Quiz 6

Ace your homework & exams now with Quizwiz!

Neurotransmitters like catecholamines (e.g., dopamine and epinephrine) have a reaction time of A) milliseconds. B) less than 10 minutes. C) 24 to 36 hours. D) 4 to 7 days.

A) milliseconds. The neurotransmitters, which control the opening of ion channels, have a reaction time of milliseconds.

When explaining the role of protein and the nine essential amino acids' needs of the body to a group of students, the nurse should emphasis that which of the following foods are complete proteins (foods that provide the essential amino acids in adequate amounts)? Select all that apply. A) Milk B) Fish C) Poultry D) Nuts E) Grains

A, B, C, D, E All of the foods listed are complete proteins from both animals and vegetable sources

Of the following patient conditions, which patients would be at risk for experiencing a thyroid problem due to a decrease in thyroxine-binding globulin (TBG)? Select all that apply. A) A 55-year-old male with cirrhosis due to alcohol abuse B) A 47-year-old female experiencing hot flashes and excess diaphoresis related to menopause C) A 75-year-old man receiving chronic glucocorticoid therapy to treat his severe chronic obstructive pulmonary disease (COPD) D) A 18-year-old female anorexia nervosa patient weighing 78 lb and has consumed no protein for the past 3 years

A, C, D A number of disease conditions and pharmacologic agents can decrease the amount of binding protein in the plasma or influence the binding of hormone. Glucocorticoid medications and systemic disease conditions such as protein malnutrition, nephritic syndrome, and cirrhosis decrease TBG concentrations.

While explaining to a group of nursing students what the function of the first mucosal layer of the lower two thirds of the esophagus, the pathophysiology instructor mentions which of the following functions? Select all that apply. A) Secretion of mucus to lubricate and protect the inner surface of the alimentary canal B) Smooth muscle cells that facilitate movement of contents of the GI tract C) Holding the organs in place and storage of fats D) Barrier to prevent the entry of pathogenic organisms E) A cushioning to protect against injury from sports or car accidents

A, D The first layer performs numerous functions. These include production of mucus that lubricates and protects the inner surface of the alimentary canal; secretion of digestive enzymes and substances that break down food; absorption of the breakdown products of digestion; and maintenance of a barrier to prevent the entry of noxious substances and pathogenic organism. The facilitation of movement of contents of the GI tract occurs in the third layer while holding the organs in place, and storage of fats occurs in the fourth layer.

When explaining about structural classifications to a group of students, the instructor discusses the peptides and proteins. They talk about small hormones and hormones as large and complex as growth hormone (GH), which has approximately how many amino acids involved? A) 50 amino acids B) 100 amino acids C) 150 amino acids D) 200 amino acids

D) 200 amino acids Growth hormone is a very large and complex protein that has approximately 200 amino acids

A middle-aged male walks into the emergency department complaining of chest pain radiating to the neck, shortness of breath, and nausea. His heart rate is 120 and BP is 94/60. The ED physician recognizes the patient is having an acute MI with decreased cardiac output. The nurse identifies the nausea to be in response to A) the patient not having a very high pain tolerance. B) hypoxia exerting a direct effect on the chemoreceptor trigger zone. C) the patient not having digested his meal completely. D) fear of having to make major lifestyle changes.

B) hypoxia exerting a direct effect on the chemoreceptor trigger zone. Hypoxia exerts a direct effect on the vomiting center (chemoreceptor trigger zone), producing nausea and vomiting. This direct effect probably accounts for the vomiting that occurs during periods of decreased cardiac output, shock, and environmental hypoxia. We are given no information about the patient's pain tolerance, when he last had a meal, or his routine lifestyle. This patient is going into shock (rapid pulse, low BP) that can result in shunting of blood away from the gut and other organs. During shock, the priority organs for oxygenation include the heart, brain, lungs, and kidneys.

A patient has recently been diagnosed with H. pylori gastritis. The nurse knows that this form of gastritis is usually treated with a combination of an antibiotic and A) antianxiety medications. B) proton pump inhibitors. C) lactulose, to reduce the blood ammonia levels. D) calcium carbonate, an antacid

B) proton pump inhibitors. H. pylori is associated with an increased risk of gastric adenocarcinoma, gastric atrophy, and peptic ulcer. It is less likely to contribute to IBD, esophagitis, or diverticular disease. Eradication of H. pylori is difficult. Treatment requires a combination therapy that includes the use of antibiotics and a proton pump inhibitor. The proton pump inhibitors have direct antimicrobial properties against H. pylori. Antianxiety medications will not kill the bacteria. H. pylori is not associated with elevated blood ammonia levels. Calcium carbonate is usually given to relieve heartburn caused by GERD

Which of the following statements most accurately captures an aspect of the basal metabolic rate (BMR)? A) The resting BMR constitutes a small fraction of the total body energy needs. B) Females tend to have a lower BMR than males due to a smaller skeletal muscle mass. C) Variations in muscle mass account for much of the differences in the BMR that exist between individuals. D) The BMR remains consistent throughout the life span.

C) Variations in muscle mass account for much of the differences in the BMR that exist between individuals. Body size and skeletal muscle mass account for most of the individual variations in BMR that exist between individuals. The resting BMR accounts for 50% to 70 % of the total body energy needs, and increased quantities of adipose tissue account for the lower average BMR in females as compared to males. BMR decreases with age.

An example of a single hormone that can exert effects in different tissues, erythropoietin, made in the kidney stimulates the bone marrow to produce A) platelets. B) natural killer cells. C) red blood cells. D) mast cells.

C) red blood cells. A characteristic of hormones is that a single hormone can exert various effects in different tissues. For example, erythropoietin, a traditional circulating hormone, is made in the kidney and stimulates erythropoiesis in the bone marrow.

Since steroid hormones are bound to protein carriers for transport, this means A) they are water soluble and circulate freely in the blood. B) they are degraded by enzymes in the blood. C) they are inactive in the bound state. D) they will be converted into a useable form by enzymes in the blood.

C) they are inactive in the bound state. Steroid hormones are bound to protein carriers for transport and are inactive in the bound state. Their activity depends on the availability of transport carriers.

A 51-year-old male professional is in the habit of consuming six to eight rum and cokes each evening after work. He assures the nurse practitioner who is performing his regular physical exam that his drinking is under control and does not have negative implications for his work or family life. How could the nurse best respond to the client's statement? A) "You are more than likely inflicting damage on your liver, but this damage would cease as soon as you quit drinking." B) "That may be the case, but you are still creating a high risk of hepatitis A or B or liver cancer." C) "In spite of that, the amount of alcohol you are drinking is likely to result first in cirrhosis and, if you continue, in hepatitis or fatty liver changes." D) "When your body has to regularly break down that much alcohol, your blood and the functional cells in your liver accumulate a lot of potentially damaging toxic byproducts."

D) "When your body has to regularly break down that much alcohol, your blood and the functional cells in your liver accumulate a lot of potentially damaging toxic byproducts." The hepatic effects of alcohol use are related to the accumulation of toxic metabolites in the hepatocytes and blood. Damage can continue even after an individual stops drinking. Specific consequences do not usually include HAV, HBV, or liver cancer. Cirrhosis represents the culmination, not the beginning, of negative hepatic effects.

After several months of persistent heartburn, a 57-year-old female client has been diagnosed with gastroesophageal reflux disease (GERD). Which of the following treatment regimens is likely to best address the woman's health problem? A) Surgical correction of the incompetent pylorus B) Antacids; avoiding positions that exacerbate reflux; a soft-textured diet C) Weight loss and administration of calcium channel blocking medications D) Proton pump inhibitors; avoiding large meals; remaining upright after meals

D) Proton pump inhibitors; avoiding large meals; remaining upright after meals Proton pump inhibitors block the final stage of gastric acid production, effectively controlling the root cause of the esophageal damage associated with GERD. The pylorus is not involved, and a soft diet is not indicated. Calcium channel-blocking drugs would not address the problem. Calcium channel blockers are primarily heart disease drugs that relax blood vessels and increase the supply of blood and oxygen to the heart while also reducing the heart's workload.

A male patient has just been diagnosed with esophageal cancer. He knew that he was losing weight and fatigued most days, but he just attributed it to aging and working. The physician recommends chemotherapy and irradiation. However, the cancer has already metastasized. The patient asks the nurse what he can expect if he agrees to the treatments. The nurse responds, A) "The therapies may shrink the cancer." B) "The doctor is prescribing treatment measures to help you swallow better." C) "These therapies will most likely cure your cancer." D) "You need to talk with your physician some more. I will page him for you."

A) "The therapies may shrink the cancer." The prognosis for people with cancer of the esophagus, although poor, has improved. Even with modern forms of therapy, the long-term survival is limited because, in many cases, the disease has already metastasized by the time the diagnosis is made. These therapies may help with food consumption, but that is not their primary purpose. Nor, will these therapies cure the cancer.

Which of the following medications used in the treatment of peptic ulcers and gastroesophageal reflux binds to H2 receptors and blocks the action of histamine on parietal cells? A) Cimetidine (Tagamet) B) Levbid (hyoscyamine) C) Lotronex (alosetron) D) Nexium (esomeprazole)

A) Cimetidine (Tagamet) Cimetidine is a H2 receptor blocker used to treat peptic ulcers and GERD. It binds to H2 receptors and blocks the action of histamine on parietal cells. Levbid and Lotronex are anticholinergics, while Nexium is a proton pump inhibitor that inhibits gastric acid secretion.

As part of the intake protocol at an eating disorders clinic, an interview precedes a physical examination. Which of the following questions would a clinician be justified in excluding from an intake interview of a 16-year-old female referred by her pediatrician for the treatment of anorexia nervosa? A) "Do you remember when your last menstrual period was? B) "Have you noticed any new hair growth on your body in the last several months?" C) "Have you had any episodes of shortness of breath in the recent past?" D) "Can you tell me about some of the habits that you have related to food in your daily routine?"

C) "Have you had any episodes of shortness of breath in the recent past?" Respiratory complications are not a noted consequence of anorexia nervosa. Amenorrhea, development of lanugo, and complex and important rituals around food preparation are common.

A female neonate has been in respiratory distress since delivery and is unresponsive to oxygen therapy. Endoscopy has confirmed a diagnosis of esophageal atresia and tracheoesophageal fistulae (EA/TEF). Which of the following explanations should the care team provide to the infant's parents? A) "We will have to perform surgery to correct the hole in her throat to make sure that she is able to swallow and breathe normally." B) "This problem will require respiratory therapy and supplementary feeding, but it will likely resolve itself over time." C) "The biggest risk that your daughter will face until this is fixed is the danger of malnutrition and dehydration." D) "The priority in our immediate treatment prior to her surgery will be pain management, as the contents of her stomach can burn her lungs."

A) "We will have to perform surgery to correct the hole in her throat to make sure that she is able to swallow and breathe normally." EA/TFE necessitate surgery and preclude both normal respiration and swallowing. Aspiration is the primary immediate risk and the priority for treatment. Although the infant will require respiratory therapy to assist with proper tube placement and ventilator maintenance, the only effective treatment is surgery. Aspiration of feeding (aspiration pneumonia) is a major complication that can occur immediately and can be life threatening. Maintaining an open airway and adequate gas exchange are the priority nursing diagnoses for this infant.

A female patient presented to her primary care physician with classic signs and symptoms of Cushing syndrome. Upon testing, it was discovered that the patient had vaginal small cell carcinoma. How can the health care providers explain her Cushing syndrome signs and symptoms to this patient? A) "Your tumor in your vagina is secreting a hormone called adrenocorticotropic hormone (ACTH), which is responsible for these signs and symptoms." B) "We are going to have to run some more tests. We think you might have a problem with your pituitary gland." C) "There is no connection between the Cushing syndrome and the vaginal carcinoma. You have two very distinct problems occurring at the same time." D) "We need to check your thyroid. Your Cushing syndrome may be caused by hypofunction of this gland."

A) "Your tumor in your vagina is secreting a hormone called adrenocorticotropic hormone (ACTH), which is responsible for these signs and symptoms." Hyperfunction is usually associated with excessive hormone production. This can result from excessive stimulation and hyperplasia of the endocrine gland or from a hormone-producing tumor. A clinical example of this phenomenon is evidenced by the case of a woman with vaginal small cell carcinoma who also presented with Cushing syndrome. After testing, it was determined that the tumor is secreting ACTH. In this situation, the cause is not related to a pituitary problem. There is a connection between Cushing syndrome and the carcinoma. The thyroid gland is not responsible for Cushing syndrome.

A 17-year-old female has announced to her family physician a desire to wholly eliminate fats from her diet. Which of the following aspects of the role of fats would underlie the physician's response to the client? A) Apart from providing energy, fats are necessary as carriers of certain vitamins and are precursors to prostaglandins. B) An extreme low-fat diet is associated with an increase in undesirable HDL cholesterol. C) Fats are a key source of dietary nitrogen, and their elimination from the diet is associated with a negative nitrogen balance. D) The total elimination of fat from the diet is associated with the development of ketosis.

A) Apart from providing energy, fats are necessary as carriers of certain vitamins and are precursors to prostaglandins. Far from being a completely undesirable component of the diet, dietary fats provide energy, function as carriers for the fat-soluble vitamins, serve as precursors of prostaglandins, and are a source of fatty acids. Low-fat diets tend to lower levels of HDL, which is a desirable form of cholesterol. Nitrogen balance is associated with protein, not fat, intake, and ketosis results from low-carbohydrate intake.

A patient being seen in the clinic has just received his routine lab results. The patient has been told that his cholesterol level is extremely elevated. The physician plans to give the patient a prescription for medication to help control this condition. Which of the following medications should the nurse prepare to educate the patient on? A) Atorvastatin (Lipitor) B) Abilify (aripiprazole) C) Pancrecarb (Pancrelipase) D) Pancrelipase (Ultrase)

A) Atorvastatin (Lipitor) The HMG-CoA reductase inhibitors or statins are used to treat high cholesterol levels by inhibiting this step in cholesterol synthesis. Abilify is for schizophrenia; Pancrecarb and Pancrelipase are pancreatic enzymes used for pancreatitis.

A nurse on a medical unit is providing care for a 37-year-old female patient who has a diagnosis of Graves disease. Which of the following treatments would the nurse most likely anticipate providing for the client? A) B-Adrenergic-blocking medications to reduce sympathetic nervous stimulation B) Administration of levothyroxine to supplement thyroid function C) Calcium channel blocking medications to reduce heart rate and cardiac risks D) Administration of somatostatin analogs to inhibit GH production

A) B-Adrenergic-blocking medications to reduce sympathetic nervous stimulation The hyperthyroidism that constitutes Graves disease can often be mitigated by the administration of B-adrenergic-blocking medications. Levothyroxine would be used to address hypothyroidism, and calcium channel blockers are not an identified treatment modality for Graves disease. Somatostatin analogs are used to treat GH excess.

A 42-year-old female client with a long-standing history of chronic nausea and vomiting but a near-insatiable appetite has had her symptoms attributed to an enzyme deficiency. Further diagnostic testing indicates that she has inadequate pancreatic enzyme levels and that her large appetite is due to a lack of enzyme control of food intake inhibition. In which of the following enzymes is the woman most likely deficient? A) Cholecystokinin B) Ghrelin C) Gastrin D) Secretin

A) Cholecystokinin Cholecystokinin is responsible for inhibiting food intake as well as stimulating pancreatic enzyme secretion. Ghrelin stimulates food intake, while gastrin stimulates gastric acid production, and secretin inhibits it.

A 28-year-old man presents with complaints of diarrhea, fecal urgency, and weight loss. His stool is light colored and malodorous, and it tends to float and be difficult to flush. He has also noted tender, red bumps on his shins and complains of pain and stiffness in his elbows and knees. Sigmoidoscopy reveals discontinuous, granulomatous lesions; no blood is detected in his stool. Which of the following diagnoses would his care team first suspect? A) Crohn disease B) Ulcerative colitis C) Diverticulitis D) Colon cancer

A) Crohn disease Crohn disease, like ulcerative colitis, causes diarrhea, fecal urgency, weight loss, and systemic symptoms such as erythema nodosum and arthritis. Unlike ulcerative colitis, it also causes steatorrhea but is not as likely to cause blood in the stool. The granulomatous "skip" lesions confirm the diagnosis of Crohn disease. Neither diverticulitis nor colon cancer would cause this combination of symptoms and signs.

A 29-year-old female has been admitted to the emergency department following a suicide attempt by overdose of acetaminophen. What changes in the client's liver and diagnostic results would the care team most likely anticipate? A) Hepatocellular necrosis evidenced by increased ALT and AST levels B) Allergic inflammation accompanied by an increase in serum IgE and basophils C) Cholestatic reaction with increased bilirubin count D) Rapid onset of hepatitis and increased GGT, ALT, and bilirubin

A) Hepatocellular necrosis evidenced by increased ALT and AST levels Acetaminophen is commonly implicated in cases of direct hepatotoxicity, a situation that is characterized by hepatocellular necrosis and increased ALT and AST levels. An allergic-type reaction is associated with idiosyncratic drug reactions, and cholestatic reactions and development of hepatitis are not noted to be associated with acetaminophen.

Chronic anxiety and stress contribute to ulcers. Which of the following effects of the sympathetic nervous system is most responsible for this effect? A) Inhibition of the actions of Brunner glands B) Overstimulation of the oxyntic glands C) Suppression of cholecystokinin D) Inflammation of the parotid glands

A) Inhibition of the actions of Brunner glands Brunner glands, which produce large amounts of alkaline mucus that protects the duodenum from acid and digestive enzymes, are strongly affected by sympathetic stimulation, which causes a marked decrease in mucus production. Where the stomach contents and secretions from the liver and pancreas enter the duodenum, deficiency of mucus can cause irritation and, potentially, ulcers. Overstimulation of the oxyntic glands is not produced by the SNS. The hormone cholecystokinin, which is thought in part to control gastric emptying, is released in response to the pH, osmolality, and fatty acid composition of the chyme. The two largest salivary glands are called parotid glands. One is located in each cheek over the jaw in front of the ears. Inflammation of one or more of these glands is called parotitis.

Testing for short stature growth hormone (GH) problems can be done by pharmacologic means. Which of the following medications can be utilized to test for a rise in GH? Select all that apply. A) Insulin B) Levodopa C) Persantine D) Dobutamine E) Sestamibi

A) Insulin B) Levodopa Diagnostic procedures for short stature include tests to exclude nonendocrine causes. If the cause is hormonal, extensive hormonal testing procedures are initiated. Tests can be performed using insulin, CHRH, levodopa, and arginine, all of which stimulate GH secretion so that GH reserve can be evaluated. Persantine, dobutamine, and sestamibi are used in cardiac stress testing. Sestamibi is also used in the testing of the parathyroid.

When examining the types of energy expenditure, which of the following statements is accurate? A) People who are more active and who fidget may have less fat gain than those with decreased nonexercise activity thermogenesis. B) Parasympathetic stimulation will cause brown fat to generate more heat than other stimulations. C) Recent research indicates that obese patients with persistent excess caloric intake have decreased sympathetic activity. D) Carbohydrate intake increases the normal metabolic rate more significantly than other nutrients.

A) People who are more active and who fidget may have less fat gain than those with decreased nonexercise activity thermogenesis. Nonexercise activity thermogenesis includes the energy expended in maintaining posture and in activities such as fidgeting. People with increased NEAT may have less fat gain than those with decreased NEAT. Sympathetic stimulation causes brown fat to generate more heat. Research shows that obese patients with excess caloric intake have increased sympathetic activity. Proteins, not carbohydrates, increase metabolic rate more significantly.

Which of the following statements best captures the essence of a second messenger in the mechanisms of the endocrine system? A) Second messengers act as the intracellular signal that responds to the presence of a hormone. B) Endocrine-producing cells must release both a hormone and a second messenger in order to exert a distant effect. C) Second messengers act to supplement hormone effects on cell receptors when the desired hormonal effect must be either increased. D) Second messengers provide an alternative pathway for endocrine effects on a cell that bypass the normal receptor pathways.

A) Second messengers act as the intracellular signal that responds to the presence of a hormone. Second messengers interact with hormones that cannot cross the cell membrane, and they mediate the ultimate effect on the cell. They are not produced by the hormone-producing cell, and they are necessary to bring about hormonal effects, not simply for increasing the intensity of the effect. They are not an alternative mechanism of effect but rather a prerequisite for certain hormonal effects on body cells.

Which of the following statements best captures an aspect of the role of hormones in the body? A) Some chemical substances can function as hormones or be integrated with the central and peripheral nervous systems. B) Hormones directly initiate many of the processes that contribute to homeostasis. C) Control of body processes is ensured by the fact that a single hormone can only exert one effect on one specific system or tissue. D) Each hormone that exists in the body is produced by only one specific endocrine gland.

A) Some chemical substances can function as hormones or be integrated with the central and peripheral nervous systems. Some chemicals, such as epinephrine, can both function as a hormone and be closely integrated with the central and peripheral nervous systems as well as the immune systems, leading to current terminology such as "neuroendocrine." Hormones modulate, but do not initiate, changes in the body, and one hormone may exert multiple effects on multiple body systems. Hormones are produced by a variety of body tissues, not solely by endocrine glands

Parents have brought their 7-year-old child into the emergency room with abdominal pain. Which of the following clinical manifestations would lead the health care team to suspect the child has appendicitis? Select all that apply. A) Tenderness in right lower quadrant with palpation B) Rebound tenderness in inguinal areas with palpation C) Redness and warmth over right lower quadrant D) Bloating and flatulence noticeable E) Urine has the smell of stool with brown coloring

A) Tenderness in right lower quadrant with palpation B) Rebound tenderness in inguinal areas with palpation Appendicitis usually has an abrupt onset. Palpation of the abdomen usually reveals a deep tenderness in the LRQ, which is confined to a small area approximately the size of the fingertip. It usually is located at approximately the site of the inflamed appendix. Rebound tenderness, which is pain that occurs when pressure is applied to the area and then released, and spasm of the overlying abdominal muscles are common. An abscess may display redness and warmth. Bloating and flatulence are usually seen with diverticular disease as is the development of fistula (vesicosigmoid), where stool is seen in the urine.

A 38-year-old male has presented to a clinic for the treatment of severe dermatitis after contact with poison ivy on a camping trip. The client has been prescribed prednisone, a corticosteroid, for the treatment of his skin condition. The client's care provider has emphasized that dosages of the drug will be gradually tapered off rather than stopped upon resolution of the symptoms. What is the most accurate rationale for this dosing protocol? A) The client's hypothalamic-pituitary-adrenal (HPA) system will require recovery time before normal function is restored. B) Steroids can induce a dependency that it best addressed with a gradual withdrawal. C) HPA function is heightened during steroid administration and must return to normal levels before the drug is completely stopped. D) Abrupt cessation of the drug can contribute to symptoms similar to Cushing syndrome.

A) The client's hypothalamic-pituitary-adrenal (HPA) system will require recovery time before normal function is restored. The suppression of the HPA system that accompanies steroid therapy requires time for a return to normal function. Dependency on the drug itself is not the rationale for tapering, and HPA function is suppressed, not heightened during therapy. Abrupt cessation can contribute to an Addison disease-like response, not Cushing syndrome.

A number of clients on a geriatric subacute medical floor of a hospital have developed foul-smelling diarrhea over the last several days, and subsequent culture of stool samples has confirmed the presence of Clostridium difficile in each case. The care team in the unit would recognize that which of the following factors likely contributed to the health problem and would anticipate which of the following treatments? A) The use of broad-spectrum antibiotics likely played a role in the development of infections, and most clients would likely receive metronidazole as a treatment. B) Genetic predisposition and the presence of the bacterium in clients' normal flora likely contributed, and treatment would consist of broad-spectrum antibiotics. C) Poor hand washing practice on the part of care providers led to the outbreak, and treatment will consist of hydration and nutritional support. D) Ingestion of contaminated food probably contributed to the infections, and corticosteroids will be needed to treat them.

A) The use of broad-spectrum antibiotics likely played a role in the development of infections, and most clients would likely receive metronidazole as a treatment. Elimination of the normal intestinal flora by broad-spectrum antibiotics commonly precedes infection by C. difficile, and metronidazole is the normal treatment. Neither genetic predisposition nor ingestion of contaminated food is a likely factor. While poor hand hygiene can spread the bacteria, treatment for C. difficile necessitates antibiotics and not simply hydration and nutritional support. The treatment of C. difficile is with antibiotics. Metronidazole is the drug of choice with vancomycin, being reserved for people who cannot tolerate metronidazole. Corticosteroids will not kill the bacterium.

A 48-year-old woman has been found to have nodules on her thyroid that must be biopsied to determine whether or not they are malignant. Which of the following imaging techniques will be most helpful to the surgeon in visualization of the nodes for fine needle aspiration? A) Ultrasound B) Magnetic resonance imaging C) Radioactive scanning using radioiodine D) Radioactive scanning using sestamibi

A) Ultrasound Thyroid ultrasound is recommended for managing thyroid nodules and can aid in visualization of the nodule for biopsy (fine needle aspiration [FNA]), which is necessary to help distinguish benign from malignant etiology. Magnetic resonance imaging is the preferred choice for pituitary and hypothalamic imaging. Isotopic imaging includes radioactive scanning of the thyroid (e.g., using radioiodine), parathyroids (e.g., using sestamibi), and adrenals (e.g., using metaiodobenzylguanidine [MIBG] to detect pheochromocytoma).

When the sympathetic nervous system is stimulated, the interstitial cells of Cajal, pacemaker cells of the GI tract, react by A) decreasing amplitude or abolishing the slow waves that control the spontaneous oscillations in membrane potentials. B) increasing the peristaltic motion of the GI tract, thereby causing explosive diarrhea. C) increasing the amount of secretions being entered into each segment of the intestinal tract. D) signaling the vagus nerve to slow down motility and increase absorption of water from the large intestine.

A) decreasing amplitude or abolishing the slow waves that control the spontaneous oscillations in membrane potentials. The interstitial cells of Cajal that are found in groups between the layers of smooth muscle tissue are hypothesized to function as the pacemakers. These cells display rhythmic, spontaneous oscillations in the membrane potentials, called slow waves, ranging in frequency from approximately 3 per minute in the stomach to 12 per minute in the duodenum. The vagus nerve responds to parasympathetic innervation. GI motility is enhanced because of increased vagal activity that could cause diarrhea.

Which of the following clients in a hospital medical unit is most clearly demonstrating the signs and symptoms of liver failure? An adult with A) low hemoglobin levels, low platelet levels, and spider angiomas present. B) blood pressure of 189/103, jaundice, and multiple thromboses. C) sudden onset of confusion, a history of alcohol abuse, and low levels of serum AST and ALT. D) ascites, fever, and recent onset of atrial fibrillation.

A) low hemoglobin levels, low platelet levels, and spider angiomas present. Anemia, thrombocytopenia, and the presence of spider angiomas are characteristic of liver failure. High blood pressure, excessive clotting, fever, and cardiac arrhythmias are not common symptoms of liver failure, and AST and ALT levels would rise, not fall.

A stroke patient is having difficulty swallowing food and beverages. The patient complains that he feels like "the food is sticking to the back of his throat." Given this information, the priority nursing interventions would be to A) make the patient "nothing per os" (NPO) and call the physician. B) feed the patient while he is sitting in an upright position. C) add a thickening agent to all of the patient's beverages. D) warrant no action since this is a normal occurrence after a stroke

A) make the patient "nothing per os" (NPO) and call the physician. People with dysphagia usually complain of choking, coughing, or an abnormal sensation of food sticking in the back of the throat or upper chest when they swallow. A neuromuscular cause involves lesions of the CNS, such as a stroke, which often involve the cranial nerves that control swallowing. Feeding in upright position is good once it is determined by swallowing evaluation that the patient can swallow food without it going into the lungs. Likewise, thickening agents help dysphagia patients after a swallow evaluation has been performed. No action could put the patient at risk for aspiration pneumonia.

Which of the following best describes the half-life of a highly protein-bound drug such as thyroxine (99% protein bound)? The half-life would be A) much longer to reduce the concentration of the hormone by one half. B) shorter because only a little of the hormone has to be used up to reduce the concentration. C) dependent on which drugs were in the blood system holding on to the hormone. D) dependent on the liver to carry the hormone to its designated target organ.

A) much longer to reduce the concentration of the hormone by one half. The half-life of a hormone—the time it takes for the body to reduce the concentration of the hormone by one half—is positively correlated with its percentage of protein binding. Thyroxine, which is more than 99% protein bound, has a half-life of 6 days, whereas aldosterone, 15% bound, has a half-life of only 25 minutes.

The physician has asked a newly diagnosed cardiac patient to begin taking omega-3 fatty acids to help prevent inflammation and blood clotting. The patient asks the nurse what types of food sources are high in omega-3 fatty acids. The nurse should educate the patient to increase his intake of which of the following items? Select all that apply. A) Salmon B) Walnuts C) Seeds D) Avocados

A, B, C All of the foods, except avocados, are rich with omega-3 fatty acids.

A teenage female has been admitted for complications resulting from bulimia nervosa. She has abused laxatives for many years and has been self-inducing vomiting since the age of 9. The nurse's admission assessment should pay close attention to which of the following complications that can arise from this disorder? Select all that apply. Assess for A) dry, cracked lips and poor skin turgor. B) missing tooth enamel and increased number of dental cavities. C) painful swallowing and stomach cramping related to reflux and esophagitis. D) fruity breath and labored, deep, gasping respirations. E) jaundice of the skin and eyes.

A, B, C Distracters A, B, and C are associated with complications of bulimia nervosa. Answer choice A relates to dehydration/fluid volume deficit; answer choice B relates to dental abnormalities associated with high acid content of the vomitus; answer choice C relates to esophagitis. Answer choice D is indicative of DKA primarily caused by undiagnosed or undertreated diabetes. Jaundice of the skin and eyes is usually associated with liver disease.

Of the following list of nursing interventions, which would be considered priority when managing a patient with life-threatening myxedematous coma? Select all that apply. A) Administer 3% sodium IV solution to increase sodium levels. B) Administer 50% dextrose to raise glucose levels. C) Place on oxygen therapy to encourage deep breathing. D) Place on a warming bed to raise body temperature. E) Administer sedatives frequently to prevent seizures.

A, B, C Myxedematous coma is a life-threatening, end-stage expression of hypothyroidism. It is characterized by coma, hypothermia, CV collapse, hypoventilation, and severe metabolic disorders that include low sodium, low glucose, and lactic acidosis. Treatment includes aggressive management of precipitating factors; supportive therapy such as management of CV status, hyponatremia, and hypoglycemia; and thyroid replacement therapy. If hypothermia is present, active rewarming is contraindicated because it may induce vasodilation and vascular collapse. Administering sedatives frequently could be harmful since the person is unable to metabolize sedatives, analgesics, and anesthetic drugs

A nurse who works in the office of an endocrinologist is orienting a new staff member. Which of the following teaching points is the nurse justified in including in the orientation? Select all that apply. A) "A bodily process can be the result of the combined effect of several different hormones from different sources." B) "A single hormone can act not only on one process or organ but often on several different locations or processes." C) "It's common for production of hormones to be far removed from the tissue where they ultimately exert their effect." D) "Sometimes hormones act locally on the area where they were produced, like in the case of paracrine and autocrine actions." E) "The regulation in homeostasis requires that hormones be absent from the body when their effect is not needed."

A, B, C, D A single hormone can exert various effects in different tissues or, conversely, a single function can be regulated by several different hormones. Hormones act both distant from their source and more locally, as in the case of autocrine and paracrine actions. Hormones are normally present at all times.

A patient has just been admitted to a nursing unit with the diagnosis of obstructive jaundice. Which of the following assessment findings would the nurse expect to see in this patient? Select all that apply. A) Clay-colored stools B) Dark urine C) Elevated conjugated bilirubin levels D) Decreased serum alkaline phosphatase levels E) Severe itching

A, B, C, E Obstructive jaundice occurs when bile flow is obstructed between the liver and the intestine. Among the causes are strictures of the bile duct, gallstones, and tumors of the bile duct or the pancreas. Conjugated bilirubin levels usually are elevated; the stools are clay colored; the urine is dark; the levels of serum alkaline phosphatase are markedly elevated; and the aminotransferase levels are slightly increased. The accumulation of bile acids in the blood leads to the development of pruritus (itching).

For several years, a 39-year-old female has been averaging two to three bottles of wine each night after her children go to sleep and has included several ounces of brandy in recent years as well. Despite negative consequences to her career and the dissolution of her marriage, her drinking has culminated in a diagnosis of cirrhosis. Which of the following physical manifestations of the health problem would her care team anticipate? Select all that apply. A) Ascites B) Anorexia C) Fever D) Bleeding tendencies E) Epigastric pain F) Obesity

A, B, D, E Ascites, anorexia, bleeding tendencies due to neurological effects, and epigastric pain are common accompaniments to cirrhosis. Ascites occurs when the amount of fluid in the peritoneal cavity is increased and is a late-stage manifestation of cirrhosis and portal hypertension. Epigastric pain is caused by liver enlargement or peritonitis. The peritoneal fluid is seeded with bacteria from the blood or lymph or from passage of bacteria through the bowel wall. Because factors V, VII, IX, and X, prothrombin, and fibrinogen are synthesized by the liver, their decline in liver disease contributes to bleeding tendencies. Fever and obesity would be less likely to exist, as the pathology is not infectious in nature and malnutrition and impaired food metabolism are common.

The patient who has been consuming a very low-calorie diet (VLCD) of 450 kcal/day should be assessed for which of the following high-risk complications? Select all that apply. A) Irregular heart rhythms B) Bone/joint inflammation C) Abdominal pain related to gallstones D) Flank pain and spasm associated with kidney sludge E) Elevated cholesterol levels

A, C VLCDs have higher risks, including abnormal heart rhythms and cholelithiasis. Anyone on this diet should be under direct supervision of a medical professional. Usually bone/joint pain/inflammation decreases with weight loss. Kidney sludge is usually related to the amount of water a person consumes.

Frustrated with his inability to lose weight despite attempting numerous fad diets, a 42-year-old male who is 5′11″ and 270 lb has visited a clinic to gain tools to achieve long-term weight loss. Which of the following statements by the clinician is most accurate? A) "Recent findings have determined that obesity is largely genetic and not preventable, but that doesn't mean we can't work together to help you lose weight and keep it off." B) "A combined approach of behavior therapy, changing your lifestyle habits, and increased physical activity gives the highest chance of long-term success." C) "By significantly changing the way you live your life, you could set and meet a goal of losing about 5% of your body weight each month." D) "Combined with regular exercise, a diet of taking in 500 to 1000 kcal/day will be the best approach."

B) "A combined approach of behavior therapy, changing your lifestyle habits, and increased physical activity gives the highest chance of long-term success." A combined approach to weight loss including diet modification, exercise, and drug therapy has been shown to be successful in the treatment of obesity. In spite of a genetic component, obesity is still considered to be preventable. A reasonable rate of weight loss should be 5% to 10% of total body weight over a 6-month period. A reduction in food intake of between 500 and 1000 kcal, not a total food intake of 500 to 1000 kcal, is often necessary in the treatment of individuals with high BMIs. Pharmacotherapy and surgery are available as adjuncts.

A 28-year-old male who is 611 tall has a diagnosis of acromegaly. The man is explaining to a curious but sympathetic coworker exactly what accounts for his extraordinary height. Which of the following explanations demonstrates a sound understanding of his health problem? A) "My pituitary gland produced a much higher than normal amount of growth hormone when I was a child." B) "A tumor in my brain threw off my hormone levels after I was finished adolescence." C) "My liver is malfunctioning and produces too many of the hormones that ultimately cause growth." D) "The high sugar levels that go along with my diabetes made my pituitary gland overproduce the hormones that cause you to grow."

B) "A tumor in my brain threw off my hormone levels after I was finished adolescence." Acromegaly is associated with adult onset and nearly always involves an adenoma. Increased GH as a child and liver dysfunction are not noted contributors to acromegaly. High levels of GH can cause overproduction of insulin and eventual diabetes, but diabetes does not itself lead to acromegaly.

Four weeks after returning from a tropical vacation, a 40-year-old man has presented to the emergency department with malaise, nausea, and "yellow eyes." Serology has confirmed a diagnosis of hepatitis A (HAV), to the shock of the client. What teaching is most appropriate for this client? A) "You can expect these symptoms to disappear after about 2 months, but you'll be a carrier of the disease indefinitely." B) "A vaccine before your trip would have prevented this, but be assured your body will rid itself of the virus in time." C) "You likely came in contact with blood or body fluids at some point, and you'll have to ensure no one is subsequently exposed to your own blood or body fluids." D) "You likely got this by way of what we call the 'fecal-oral' route; you will have chronic hepatitis now, but the symptoms can be controlled with medication."

B) "A vaccine before your trip would have prevented this, but be assured your body will rid itself of the virus in time." HAV is normally self-limiting and does not result in chronic hepatitis or carrier status. A vaccine is available, and the fecal-oral route of transmission, rather than contact with blood and body fluids, is typical.

A 40-year-old man who uses heroin intravenously was diagnosed with hepatitis C (HCV) 1 year ago and is now considered to have chronic viral hepatitis. Which of the following statements by the client to his care provider would warrant correction? A) "I know the medications to treat this aren't fantastic, but at least there are some options for controlling the virus." B) "It's at least a bit reassuring that my liver isn't undergoing damage when I'm not experiencing symptoms." C) "Even though I'm sick, at least I won't feel sick most of the time." D) "I'm not looking forward to all the side effects of the drug treatments for my HCV, but I hope I don't end up needing a liver transplant."

B) "It's at least a bit reassuring that my liver isn't undergoing damage when I'm not experiencing symptoms." Liver damage persists both during symptomatic and asymptomatic periods of acute viral hepatitis. Medications do exist for treatment of the disease, and asymptomatic periods are more common than symptomatic episodes. Side effects of drug treatment are common, and transplant is a potential end-stage treatment option.

A gastroenterologist is teaching a group of medical students about the enteric nervous system in preparation for a consult on client who has suffered a spinal cord injury. Which of the physician's teaching points is most accurate? A) "The myenteric plexus is responsible for controlling the function of each segment of the intestinal tract." B) "The enteric nervous system is made up of the myenteric and submucosal plexuses; these are located in the wall of the GI tract." C) "Sympathetic innervation of much of the GI tract occurs by way of the vagus nerve." D) "Parasympathetic stimulation blocks the release of the excitatory neuromediators and inhibits GI motility."

B) "The enteric nervous system is made up of the myenteric and submucosal plexuses; these are located in the wall of the GI tract." The enteric nervous system consists of the myenteric and submucosal plexuses, which are located within the wall of the gastrointestinal tract. The myenteric plexus is responsible for controlling overall function along the entire length of the gut, while the vagus nerve provides parasympathetic, not sympathetic, innervation. Sympathetic simulation lessens excitatory neuromediators and inhibits GI motility.

A dietitian is working with a morbidly obese client in an effort to facilitate weight loss. Which of the dietitian's following teaching points about the nature of adipose tissue should be included in the client education? A) "Our ultimate goal is going to be eventually rid your body of adipose tissue or fat." B) "Your fat cells can be considered to be one large energy storage organ that also has a role in hormone production." C) "We ideally would like to maximize your levels of brown fat and minimize those of white fat." D) "Obesity is normally the result of the number of 'pre-fat' cells an individual is born with."

B) "Your fat cells can be considered to be one large energy storage organ that also has a role in hormone production." Fat cells are collectively considered a large body organ that is metabolically active in the uptake, synthesis, storage, and mobilization of lipids, which are the main source of stored fuel for the body; the role of adipose tissue as an endocrine organ has also been recently elucidated. It is neither a desirable nor reasonable goal to entirely rid the body of fat, given the key roles it plays in homeostasis, and brown fat is not common in postnatal life. Preadipocytes have been shown to play a role in obesity, but the condition is still primarily a consequence of energy intake exceeding output.

Which of the following clients is most clearly displaying the signs and symptoms of irritable bowel disease (IBD)? A) A 32-year-old mother who complains of intermittent abdominal pain that is worse during her menstrual period B) A 51-year-old male who states that his stomach pain is in his lower abdomen, "comes and goes," and "feels more like a cramp than a dull ache" C) A 44-year-old man who works the evening shift at a factory and who states that his lower abdominal pain is much worse at night than during the day D) A 24-year-old man who has a stressful job but whose diarrhea and cramping do not worsen during periods of high stress

B) A 51-year-old male who states that his stomach pain is in his lower abdomen, "comes and goes," and "feels more like a cramp than a dull ache" IBD is commonly manifested as intermittent lower abdominal pain that feels like cramping. Defecation normally relieves the pain, and symptoms are normally not present at night or during sleep. Stress commonly exacerbates symptoms.

Which of the following statements about types of obesity is most accurate? A) Upper body obesity is often referred to as being shaped like a "pear." B) A waist-hip ratio greater than 1.0 in men can be interpreted to mean upper body obesity. C) Waist circumference is a measurement of subcutaneous abdominal adipose tissue but not intra-abdominal adipose tissue. D) A waist circumference of 40 inches or less in women is considered normal and therefore not associated with increased health risk.

B) A waist-hip ratio greater than 1.0 in men can be interpreted to mean upper body obesity. A waist-hip ratio greater than 1.0 in men and 0.8 in women indicates upper body or central (abdominal) obesity. Upper body obesity is referred to as being shaped like an apple. Waist circumference measures mostly subcutaneous and intra-abdominal adipose tissue. In males, a waist circumference greater than 40 inches (or 35 inches in females) is associated with increased health risk.

A patient exhibiting problems with his or her thyroid has been scheduled for a radioactive scan. From the following list of patients, what would the nurse question as to whether this would be a safe procedure for this patient? A) An adult patient having an episode of wheezing from allergies B) A young female patient who has been trying to get pregnant C) A middle-aged male patient with uncontrolled type 2 diabetes mellitus D) An elderly patient who has a history of aortic stenosis

B) A young female patient who has been trying to get pregnant Radioactive iodine therapy is contraindicated in pregnant women because 131I crosses the placenta and can adversely affect the fetal thyroid gland. The other clients would have no contraindication to the substance.

Following a meal, a woman's blood glucose level has increased. In addition, her pancreas has increased the amount of insulin produced and released. Which of the following phenomena has occurred? A) Increased hormone level according to a negative feedback mechanism B) Adjustment according to the level of the substance a hormone regulates C) Hormone production and release via the positive feedback cycle D) Hypothalamic-pituitary control of hormone levels

B) Adjustment according to the level of the substance a hormone regulates The level of some hormones is adjusted according to the amount of the substance that they control. In this case, insulin controls glucose levels and would increase in response to the increase in serum glucose that follows a meal. This differs from a negative feedback cycle in which a simple decrease in a hormone level stimulates production and/or release of that hormone. Positive feedback and hypothalamic-pituitary control are not evident in this situation.

A 43-year-old male client has presented to the emergency department with vomiting that he claims is of a sudden onset. The client also states that the emesis has often contained frank blood in the hours prior to admission. His vital signs are stable with temperature 98.3°F, pulse 88, BP 140/87, and respiratory rate 18. Which of the following potential contributing factors would the health care team suspect first? A) Overuse of antacids B) Alcohol consumption C) Staphylococcal enterotoxins D) Effects of Helicobacter pylori

B) Alcohol consumption Acute gastritis associated with alcohol use is characterized by intermittent vomiting and the possibility of hematemesis. Aspirin and H. pylori do not normally cause such an acute symptom onset, and infectious organisms do not normally cause bleeding of the stomach lining. A combination of calcium carbonate and magnesium is commonly found in antacids. Overdose of antacids can result in irregular heartbeat, poor balance, shallow, rapid breathing and stupor (lack of alertness).

The mother of a 19-week-old infant has brought her baby in for assessment to a pediatrician because of the baby's persistent weight loss and diarrhea. An intestinal biopsy has confirmed a diagnosis of celiac disease, and the child's mother is anxious to know what caused the disease. Which of the following aspects of the etiology of celiac disease would underlie the explanation that the physician provides? A) Bacterial or chemical invasion of the peritoneum leads to decreased nutrient absorption and transport. B) An inappropriate T-cell-mediated response results in increased levels of antibodies and an inflammatory response. C) Neurogenic or muscular inhibition of peristalsis results in inappropriate motility of ingested food in the lower small intestine and the colon. D) Inability to process or absorb the fat content of breast milk results in malnutrition and deficiency of fat-soluble vitamins.

B) An inappropriate T-cell-mediated response results in increased levels of antibodies and an inflammatory response. Celiac disease is rooted in an inappropriate immune response that initiates an inflammatory response, resulting in loss of absorptive villi. Bacterial or chemical invasion of the peritoneum is associated with peritonitis, while inhibition of peristalsis is associated with obstructions. An inability to process or absorb fat is associated with malabsorption syndrome.

A 53-year-old woman with a history of chronic alcohol abuse but without visible jaundice comes to the clinic complaining of nausea and weakness. She admits to taking acetaminophen for persistent headaches but denies exceeding the recommended daily dose; she has not taken any other medications. She is suspected of having acetaminophen toxicity. Which of the following diagnostic test findings would implicate a different cause of her symptoms? A) Normal serum acetaminophen level B) Elevated serum HBsAg level C) Evidence of steatosis on liver biopsy tissue sample D) Hypoglycemia

B) Elevated serum HBsAg level The presence of HBsAg would suggest that this woman is in the prodromal phase of hepatitis B infection. A normal serum acetaminophen level does not preclude toxicity if the drug is taken over a period of time. Steatosis is fatty infiltration of the liver. Steatosis is often but not exclusively an early histological feature of alcoholic liver disease (therefore, with chronic alcohol abuse, the nurse should expect the patient may have steatosis). Drinking heavily without eating can block your liver from releasing stored glucose into your bloodstream, causing hypoglycemia.

Which of the following measures would likely be rejected as part of a first-line weight loss plan for a client with a BMI of 30.2, type 2 diabetes, and hypertension? A) Prescription drug therapy B) Gastric bypass C) Calorie reduction by 500 to 1000 kcal/day D) 30 minutes or more of moderate-intensity activity at least 3 days per week

B) Gastric bypass Weight loss surgery should be limited to individuals with a BMI greater than 40 or those with a BMI greater than 35 with comorbid conditions in whom medical therapy has failed. Calorie reduction is appropriate for anyone with a BMI of 25 to 29.9 plus two risk factors (in this case, type 2 diabetes and hypertension). Prescription drug therapy can be considered for those with a BMI of 30 or more, and increased physical activity, although it does not lead to significant weight loss, helps prevent further weight gain and reduces cardiovascular and diabetes risk beyond that achieved by weight loss alone.

A male patient comes to the clinic asking to speak to a health care provider privately. He reveals that he had shared a needle/syringe with a prostitute (shooting up cocaine) and then had unprotected sex. Upon questioning, it was revealed that the patient had not had any immunization for hepatitis B. Which of the following medications would the nurse anticipate administering today to this patient? A) Tenofovir disoproxil fumarate plus emtricitabine B) Hepatitis B immunoglobulin (HBIG) C) Hepatitis C immunoglobulin (HCIG) D) Hepatitis A vaccine

B) Hepatitis B immunoglobulin (HBIG) HBIG is used as an adjunct to hepatitis B vaccine for postexposure immunoprophylaxis to prevent HBV infections in high-risk populations. Tenofovir disoproxil fumarate plus emtricitabine is for HIV preexposure coverage. There is no hepatitis C vaccine available on the market yet. Hepatitis A vaccine is for HAV, which is usually spread from fecal-oral routes.

Following the identification of low blood levels of cortisol and low 24-hour urinary free cortisol, a 51-year-old female client has been diagnosed with a primary adrenal cortical insufficiency. Which of the following health consequences would be attributable to her low levels of cortisol? A) Visible exophthalmos B) Impaired immunological and inflammatory response C) Diminished secondary sex characteristics D) Insufficient regulation of serum potassium and sodium levels

B) Impaired immunological and inflammatory response Cortisol plays a central role in the normal functioning of the immune response and inflammation. Exophthalmos is associated with Graves disease, and secondary sex characteristics are functions of adrenal androgens. Potassium and sodium are regulated by mineralocorticoids.

A 22-year-old student has developed a fever and diarrhea while on a backpacking trip in Southeast Asia. His oral temperature is 101.4°F. The diarrhea is bloody, frequent, and small in volume. These clinical manifestations are sufficiently distressing that he is visiting a local medical clinic in the area. Which of the following diagnoses best characterizes this health problem? A) Noninflammatory diarrhea B) Inflammatory diarrhea C) Factitious diarrhea D) Secretory diarrhea

B) Inflammatory diarrhea Inflammatory diarrhea is often characterized by small-volume diarrhea that is bloody and accompanied by a fever. Noninflammatory diarrhea is normally larger in volume and not bloody. Factitious diarrhea is normally attributable to laxative use, and secretory diarrhea is associated with increased secretory processes of the bowel; neither is likely to produce bloody stool.

To reduce hepatic blood flow and decrease portal pressures in persons with cirrhosis, the nurse should be prepared to administer which of the following medications? A) Bevacizumab, an angiogenesis inhibitor B) Octreotide, a long-acting synthetic analog of somatostatin C) Filgrastim, granulocyte colony-stimulating factor (G-CSF) analog D) Diltiazem (Cardizem), a calcium channel blocker

B) Octreotide, a long-acting synthetic analog of somatostatin Ocetrotide, a long-acting synthetic analog of somatostatin, reduces splanchnic and hepatic blood flow and portal pressures in persons with cirrhosis. Bevacizumab was the first angiogenesis inhibitor that was shown to slow tumor growth and, more importantly, to extend the lives of patients with some cancers. Filgrastim rapidly reverses neutropenia and maintains normal ANC in patients with HIV infection. Diltiazem is used to treat a variety of conditions, such as high blood pressure, migraines, and Raynaud disease.

Following destruction of the pituitary gland, ACTH stimulation stops. Without ACTH to stimulate the adrenal glands, the adrenals' production of cortisol drops. This is an example of which type of endocrine disorder? A) Primary B) Secondary C) Tertiary D) Somatic

B) Secondary In secondary disorders of endocrine function, the target gland is essentially normal, but defective levels of stimulating hormones or releasing factors from the pituitary system alter its function.

Which of the following statements best captures the role of the adrenal cortex in maintaining homeostasis? A) The adrenal cortex is responsible for the production of epinephrine and norepinephrine that are part of the sympathetic nervous system. B) The adrenal cortical hormones are primarily steroids and sex hormones. C) Redundant, secondary production of adrenal cortical hormones can compensate for the loss of the adrenal glands. D) Normal sexual function is dependent on adequate adrenal cortical function

B) The adrenal cortical hormones are primarily steroids and sex hormones. The adrenal cortex is responsible for secreting three types of hormones: the glucocorticoids, the mineralocorticoids, and the adrenal androgens. The adrenal medulla produces epinephrine and norepinephrine, and there are no alternate production sites for adrenal cortical hormones. The adrenal androgens are least responsible for normal sexual function.

During the follicular stage of menstruation, increased estradiol production causes an increase in FSH production. This increase in FSH production by the anterior pituitary gland will have what effect on the follicle? A) The follicle will continue to grow until it can no longer stay in its membrane. B) The follicle will die, which results in a fall of FSH. C) The follicle will continue to grow and produce estradiol. D) The follicle will secrete additional hormones to attract swimming sperm.

B) The follicle will die, which results in a fall of FSH. In positive feedback control, rising levels of a hormone cause another gland to release a hormone that is stimulating to the first. Increased estradiol production during the follicular stage of the menstrual cycle causes increased FSH production by the anterior pituitary gland. This stimulates further increases in estradiol levels until the demise of the follicle

A middle-aged woman has acromegaly as a result of a pituitary adenoma that was found and removed when she was a teenager. The physician is suspecting that the tumor has returned and has ordered a diagnostic work-up. A glucose load is ordered. If the tumor has returned, the nurse would expect which of the following results? A) The glucoses load will suppress GH level. B) The growth hormone level will not be suppressed following glucose load. C) The glucose load will raise her serum glucose level to the point of requiring insulin. D) There will be no change in the serum growth hormone level following the glucose load.

B) The growth hormone level will not be suppressed following glucose load. When a GH-secreting tumor is suspected, the GH response to a glucose load is measured as part of the diagnostic workup. Normally, a glucose load would suppress GH levels. However, in adults with GH-secreting tumors (acromegaly), GH levels are not suppressed (and paradoxically increase in 50% of cases) to a glucose load.

Following a long history of fatigue, weakness, and poor appetite, a 39-year-old male has been diagnosed with hypopituitarism. Which of the following clinical findings would most likely cause his care team to suspect that the man has an additional endocrine disorder from a different source? A) The man has a low sperm count and has been unable to have children. B) The man has a chronic platelet deficiency and is occasionally anemic. C) The client is 5'2" tall and was consistently short for his age as a child. D) The man displays the signs and symptoms of hypothyroidism.

B) The man has a chronic platelet deficiency and is occasionally anemic. Low platelets and low hemoglobin are unlikely to be a manifestation of hypopituitarism. A low sperm count, small stature, and hypothyroidism are all noted manifestations of pituitary hypofunction.

Which of the following statements best captures an aspect of the function of the hypothalamic-pituitary-adrenal (HPA) system? A) Adrenocorticotropic hormone (ACTH) released by the hypothalamus controls to release of cortisol. B) The pituitary gland communicates with the adrenal cortex through the release of ACTH. C) The adrenal cortex receives corticotrophin-releasing hormone (CRH) and in turn releases cortisol. D) The pituitary gland causes a release of CRH from the hypothalamus, which promotes hormone release from the adrenal cortex.

B) The pituitary gland communicates with the adrenal cortex through the release of ACTH. ACTH mediates between the anterior pituitary gland and the adrenal cortex in the HPA system. ACTH is released by the pituitary, not the hypothalamus, and CRH acts on the pituitary, not the adrenal cortex. CRH flows from the hypothalamus to the pituitary, not vice versa.

Which of the following statements best captures an aspect of the process of fat digestion and absorption? A) Ingested triglycerides are broken down into absorbable form by gastric lipase. B) Triglycerides are digested with the aid of bile salts. C) Long-chain fatty acids are absorbed directly into the portal blood. D) Stool is not excreted until all fat is absorbed

B) Triglycerides are digested with the aid of bile salts. After breakdown, by pancreatic lipase, triglycerides are absorbed primarily in the upper jejunum. Long-chain fatty acids are absorbed less easily than medium-chain triglycerides, while stool often contains a certain amount of fat.

A client with a new diagnosis of an endocrine disorder is unclear how the body can control the levels of different hormones over time. Which of the following statements most accurately underlies the dominant regulation process of hormone levels in the body? A) A positive feedback cycle ensures that stable levels of hormones exist in the body over time. B) With input from various sensors, hormone production and release are adjusted based on existing hormone levels. C) The hypothalamus ensures that hormone levels correspond accurately to the diurnal cycle. D) The pituitary gland is genetically programmed to stimulate and inhibit hormone production and/or release based on the needs at different points in the life cycle.

B) With input from various sensors, hormone production and release are adjusted based on existing hormone levels. Most hormone levels are controlled by way of a negative feedback cycle, in which low levels stimulate production and/or release. A positive feedback cycle would not achieve this effect. While some hormones are released on a diurnal schedule, the dominant form of hormone regulation in the body is that of negative feedback. Hormone release is not predetermined by the pituitary gland.

A mom asks her neighbor, a nurse, why every time she takes her daughter (10 years old) out for ice cream she comes home with a stomachache and then experiences a bout of diarrhea. The nurse is thinking that this girl is experiencing A) anxiety about increasing too many calories. B) a deficiency of lactase. C) gallbladder disease. D) premature peptic ulcer formation.

B) a deficiency of lactase. People with a deficiency of lactase, the enzyme that breaks down lactose, experience diarrhea when they drink milk or eat dairy products. Doubtful the child is anxious about the calories in ice cream. Gallbladder disease s/s usually occur in relation to high saturated fat intake. Peptic ulcer s/s include bloating, vomiting blood, foul coffee ground stools, etc.

A 68-year-old African American man who has smoked for at least 50 years reports that lately he feels as though food is getting stuck in his throat. At first, this was a problem just with dry food, but now his morning oatmeal is getting "stuck." On questioning, he reports drinking at least three alcoholic beverages nearly every day. His problem is most likely A) achalasia. B) squamous cell carcinoma of the esophagus. C) dysphagia secondary to scleroderma. D) gastrointestinal reflux disease.

B) squamous cell carcinoma of the esophagus. Squamous cell carcinoma of the esophagus is the seventh leading cause of cancer death among men, particularly black men; mean age at diagnosis is 67 years. Alcohol and tobacco use are the main risk factors for this cancer, and dysphagia is a common presenting complaint. An esophageal motility disorder involves the smooth muscle layer of the esophagus and the lower esophageal splincter (LES). Achalasia is characterized by difficulty swallowing and regurgitation. GERD (gastroesophageal reflux disease) is a condition that causes the esophagus to become irritated and inflamed. Clients with GERD usually feel a burning in the chest or throat called heartburn. Sometimes, they taste stomach fluid in the back of the mouth.

An infant born with congenital hypothyroidism and has not sought care from any health care provider is likely to develop which of the following complications? Select all that apply. A) Deformed joints and bone spurs B) Impaired physical growth C) Mental retardation D) Loss of fine motor control and arthritis E) Down syndrome

B, C Thyroid hormone is essential for normal growth and brain development, almost half of which occurs during the first 6 months of life. If untreated, congenital hypothyroidism causes mental retardation and impairs physical growth. Down syndrome is a congenital birth defect and not caused by hypothyroidism.

A 40-year-old female has been categorized as being obese, with a body mass index (BMI) of 33.2. Which of the following health problems place the client at a significantly increased risk for when compared with individuals with a BMI below 25? Select all that apply. A) Cardiac arrhythmias B) Osteoarthritis C) Multiple sclerosis D) Atelectasis E) Gallbladder disease F) Insulin resistance

B, E, F Obesity is associated with significantly increased risk for osteoarthritis due to bone and joint stress. Insulin resistance and gallbladder disease are also identified consequences of obesity. Cardiac arrhythmias are less likely to result directly from obesity, given their etiology rooted in electrical conductivity. Multiple sclerosis and other neurological effects are also unlikely, and atelectasis is not commonly a direct effect of high levels of body fat.

A patient with pancreatic cancer is admitted for portal hypertension in which he is symptomatic with ascites. Following paracentesis and removal of 7.5 L of ascitic fluid, the nurse should anticipate that the physician will order which of the following medications to assist in maintaining an effective circulating fluid volume? A) Bumetanide (Bumex) B) Furosemide (Lasix) C) Albumin (human) 5% D) Epogen (epoetin alfa)

C) Albumin (human) 5% Complications of portal hypertension include ascites, splenomegaly, and hepatic encephalopathy. Following paracentesis, to remove ascitic fluid, a volume expander such as albumin is usually administered to maintain the effective circulating volume. Lasix and Bumex are diuretics that decrease circulating fluid volume. Epogen stimulates red blood cell production and ultimately increases O2-carrying capacity.

Which of the following hormones are derivatives of cholesterol? A) Epinephrine and norepinephrine B) Insulin and glucagon C) Aldosterone and testosterone D) Eicosanoids and retinoids

C) Aldosterone and testosterone Steroids such as aldosterone and testosterone are a classification of hormones that are derived from cholesterol. Epinephrine and norepinephrine are both amino acids, while insulin and glucagon are classified among peptides, polypeptides, proteins, and glycoproteins. Eicosanoids and retinoids consist of fatty acid compounds.

Following several days of intermittent upper right quadrant pain, a 29-year-old obese, Native American woman has been diagnosed with cholelithiasis. The nurse at the clinic has taught the client about the pathophysiology and contributing factors to her health problem, as well as some of the likely treatment options. Which of the following statements by the client demonstrates a sound understanding of her diagnosis? A) "All in all, I guess this is a result of the fact that I've been eating a diet too high in cholesterol for too long." B) "Several factors like my genetics and gender may have contributed to this, but I'm glad that medications can cure it." C) "This explains why my skin was yellow-tinged lately and why I had those pains that spread to my upper back and right shoulder." D) "I suppose the fever and vomiting I had this week was probably a sign of my gallstones too."

C) "This explains why my skin was yellow-tinged lately and why I had those pains that spread to my upper back and right shoulder." Gallstones can be caused by abnormalities in the composition of bile (increased cholesterol) and stasis of bile. The formation of cholesterol stones is associated with obesity and occurs more frequently in women. These factors cause the liver to excrete more cholesterol into the bile. Estrogen reduces the synthesis of bile acid in women. Cholesterol stones are extremely common in Native Americans. Jaundice and pain that radiates to the upper back and right shoulder are noted signs and symptoms of cholelithiasis. While cholesterol is a key element in the formation of gallstones, the particular amount ingested in the diet is not central to the development of the problem. Surgery, not medication, is the normal treatment modality, and fever and nausea are more closely associated with cholecystitis rather than cholelithiasis.

A 71-year-old male has been recently diagnosed with a stage III tumor of colorectal cancer and is attempting to increase his knowledge base of his diagnosis. Which of the following statements about colorectal cancer demonstrates a sound understanding of the disease? A) "If accurate screening test for this type of cancer existed, it could likely have been caught earlier." B) "The NSAIDs and aspirin that I've been taking for many years probably contributing to me getting cancer." C) "While diet is thought to play a role in the development of colorectal cancer, the ultimate causes are largely unknown." D) "A large majority of patients who have my type of colon cancer survive to live many more years."

C) "While diet is thought to play a role in the development of colorectal cancer, the ultimate causes are largely unknown."

A 70-year-old male with a 40 pack-year history of smoking and long-standing non-insulin-dependent diabetes has been diagnosed with pancreatic cancer. Which of the following teaching points should the physician provide? A) "While this is indeed serious, you should know that you have a good chance of beating this disease with appropriate treatment." B) "Most likely your pattern of high alcohol intake over the years contributed to your cancer." C) "You will likely be facing surgery in the near future, but know that this is very unlikely to eliminate your cancer." D) "I know it may seem trivial at this point, but the levels of pain that accompany cancer of the pancreas are normally quite low."

C) "You will likely be facing surgery in the near future, but know that this is very unlikely to eliminate your cancer." Surgery is the most common treatment modality for pancreatic cancer, but it does not commonly result in the resolution of the disease. Pancreatic cancer has a very poor prognosis, and alcohol is not a common contributor. Pancreatic cancer is noted to be exceptionally painful.

Following a history of gastric pain and an endoscopy, a client has been diagnosed with a duodenal peptic ulcer. Which of the following teaching points should his caregiver provide? A) "While your diet most certainly contributed to this problem, the good news is that changing your diet can help solve it." B) "Ulcers like yours do not penetrate all layers of the stomach or duodenum, so you don't have to worry about losing too much blood." C) "Your family history, your smoking history, and NSAID use may all have contributed to this problem." D) "While there aren't really any effective medications for these ulcers, changes in lifestyle can keep them well controlled."

C) "Your family history, your smoking history, and NSAID use may all have contributed to this problem." Family history, NSAID use, and smoking have all been identified as contributing factors in the development of peptic ulcers. Diet therapy has not been shown to be effective, and duodenal peptic ulcers are more common than the gastric variant. Perforation occurs when an ulcer erodes through all layers of the stomach or duodenum wall. When perforation occurs in older adults, their mortality is significantly increased. Effective medication regimens are available with antacids, H2-receptor antagonists or proton pump inhibitors being the most common medications used.

A patient has just been diagnosed with pernicious anemia. The patient asks the nurse why his body is not able to absorb vitamin B12. The nurse responds, A) "You have too many ulcers in your stomach." B) "More than likely, you were born with deficient vitamin B12 stores." C) "Your stomach is not secreting a substance known as intrinsic factor, which is needed to absorb vitamin B12." D) "Your daily intake of high saturated fats is interfering with the stomach's ability to absorb the nutrients that it needs, especially vitamin B12."

C) "Your stomach is not secreting a substance known as intrinsic factor, which is needed to absorb vitamin B12." Vitamin B12 is not absorbed in the absence of intrinsic factor, which is secreted by the parietal cells of the stomach. Ulcers in the stomach do not cause pernicious anemia. This is not a congenital problem, nor does the intake of fats interfere with the secretion of intrinsic factor.

When counseling a group of overweight individuals, the nurse should stress that during parties, the oxidation of alcohol provides how many kilocalorie/gram to one's diet? A) 4 kcal/g B) 5 kcal/g C) 7 kcal/g D) 9 kcal/g

C) 7 kcal/g The oxidation of proteins provides 4 kcal/g; fats, 9 kcal/g; carbohydrates, 4 kcal/g; and alcohol, 7 kcal/g.

The mother of 6-year-old male and female fraternal twins has brought her son to see a pediatrician because he is nearly 4 inches shorter than his sister. Which of the following phenomena would the physician most likely suspect as contributing factor to the boy's short stature? A) Genetic short stature B) Lack of IGF receptors in epiphyseal long bones C) A shortage of hypothalamic GHRH production D) Excess insulin production resulting in chronically low blood glucose levels

C) A shortage of hypothalamic GHRH production Inadequate levels of hypothalamic GHRH will result in adequate production but inadequate release of GH by the pituitary. Genetic short stature is less likely given the disparity between his height and his twin's, and a shortage of IGF receptors is not a noted pathology. While poorly controlled diabetes can contribute to short stature, excess insulin production is not a likely factor.

Following a 14-day course of broad-spectrum antibiotics for the treatment of sepsis, a 60-year-old woman has developed watery diarrhea. Her care team attributes this to likely elimination of normal intestinal flora by the antibiotics. What other phenomena is most likely accompanying her low levels of normal flora? A) Decreased mineral and nutrient absorption; decreased carbohydrate metabolism B) Decreased pH of the stomach; increased pH of the lower GI tract C) Decreased fermentation of undigestible dietary residue; decreased vitamin absorption D) Proliferation of vitamin K; lower GI bleeding

C) Decreased fermentation of undigestible dietary residue; decreased vitamin absorption Central among the functions of normal intestinal flora are the fermentation of dietary components that are not digestible and the facilitation of vitamin absorption. Mineral and nutrient absorption as well as carbohydrate metabolism are less likely to be affected, while vitamin K production would likely decrease somewhat. pH is unlikely to be affected.

Parents of a 20-month-old infant report that he refuses food or eats poorly and that he grimaces when he swallows. He also is irritable and cries a lot. The mother is worried that he ate something inappropriate this morning, because he vomited something that looked like coffee grounds. Which of the following health problems would the care team first suspect? A) Rotavirus infection B) Appendicitis C) Esophagitis from gastrointestinal reflux D) Hirschsprung disease

C) Esophagitis from gastrointestinal reflux Esophagitis secondary to reflux can cause feeding problems, early satiety, and hematemesis. Infants may demonstrate signs of pain when swallowing and may be irritable and cry frequently. Rotavirus causes diarrhea and vomiting, but not the other symptoms. Appendicitis is inflammation of the appendix. Appendicitis usually starts with the main symptom of pain around the navel that moves to the lower right abdomen. Hirschsprung disease is a blockage of the large intestine due to improper muscle movement in the bowel. It is a congenital condition, which means it is present from birth. In Hirschsprung disease, the nerves are missing from a part of the bowel. One primary s/s is a failure to pass meconium shortly after birth.

A 38-year-old woman takes clomiphene, an infertility drug that works by competing with, and thereby blocking, cellular receptors for estrogen. Which of the following statements is most likely to be true of this client? A) Receptors for all other steroid hormones will also be blocked. B) Up-regulation will increase the number of estrogen receptors on each target cell. C) Estrogen will continue to pass freely through the cellular membranes. D) Laboratory tests will reveal an increase in cyclic adenosine monophosphate (cAMP) levels.

C) Estrogen will continue to pass freely through the cellular membranes. Because estrogen is a steroid hormone, its receptors in target cells are located inside the cell membrane, and their blockage does not affect the movement of the hormone into and out of the cell. Receptors are specific for each hormone, so no hormones other than estrogen will be blocked. Up-regulation occurs when hormone levels are decreased, and in this case, the estrogen level will increase. Second messengers, such as cAMP, are only activated by peptide hormones and catecholamines.

A science teacher is talking to a group of fifth graders about the role of "spit." During the course of the discussion, the teacher asks the students which of the following are functions of saliva? A) Good protection device if someone is being attacked B) Will have more saliva production when anxious, such as right before a test C) Has antibacterial action to help keep the mouth clean D) Secretes acid to chemically break down fatty foods like French fries E) Secretes the enzyme pepsin

C) Has antibacterial action to help keep the mouth clean Saliva has three functions. The first is to protect and lubricate. The second is to provide antimicrobial protection. The third is to initiate digestion of starches by secreting enzymes. If anxious, SNS activity causes dry mouth. Saliva begins breaking down starches (carbohydrates), not fatty foods. The chief cells secrete pepsinogen, which is converted into pepsin, the enzyme that breaks down protein. This occurs in the stomach.

A nurse practitioner is providing care for a male client with a long-standing hiatal hernia. Which of the following statements most accurately captures an aspect of the pathophysiology of hiatal hernias? A) Paraesophageal hiatal hernias are common and are normally not treated if the client is asymptomatic. B) The root causes of hiatal hernias are normally treatable with medication. C) If esophageal acid clearance is impaired, esophagitis can result. D) An incompetent pyloric sphincter and high-fat diet are commonly implicated in the development of hiatal hernias.

C) If esophageal acid clearance is impaired, esophagitis can result. Erosive esophagitis can be a complication of hiatal hernias if esophageal acid clearance is significantly impaired. Paraesophageal hiatal hernias are more serious than the sliding variety and require treatment. The root cause of hiatal hernias, herniation of the stomach through the diaphragm, is not normally amenable to treatment with medication. The pyloric sphincter is not associated with hiatal hernias

A 20-year-old male who is addicted to crystal methamphetamine has been admitted to a hospital with a diagnosis of protein-calorie malnutrition after many months of inadequate food intake. Which of the following treatment plans would the care team most likely favor? A) Intravenous infusion of albumin coupled with vitamin supplementation B) Total parenteral nutrition C) Incrementally feeding combined with vitamin and mineral supplementation D) Rapid administration of normal saline and carbohydrates

C) Incrementally feeding combined with vitamin and mineral supplementation Slow administration of protein and calories combined with mineral and vitamin supplementation is important in the treatment of protein-calorie malnutrition. Albumin transfusions and total parenteral feeding would likely not be necessary, and rapid administration of fluids and carbohydrates may precipitate congestive heart failure.

A 51-year-old woman has been experiencing signs and symptoms of perimenopause and has sought help from her family physician. A deficiency in estrogen levels has been determined to be a contributing factor. Which of the following phenomena could potentially underlie the woman's health problem? A) Sufficient synthesis of estrogen but inadequate vesicle-mediated release B) Inadequate synthesis in the rough endoplasmic reticulum of her ovarian cells C) Insufficient estrogen production within the smooth endoplasmic reticulum of the relevant cells D) A lack of prohormone precursors needed for estrogen synthesis and release

C) Insufficient estrogen production within the smooth endoplasmic reticulum of the relevant cells Steroids such as estrogen are produced in the smooth endoplasmic reticulum. Synthesis and release are not separate processes as in the case of peptide hormones, and prohormones are associated with peptide, polypeptide, and protein hormones.

The instructor asks a group of nursing students to explain the function of the omentum. The students will respond based on which pathophysiologic principle? A) It holds organs in place. B) It attaches the jejunum and ileum to the abdominal wall. C) It has lots of mobility and moves around in the peritoneal cavity with peristaltic movements. D) It is mainly there to prevent any noxious substance from inner into the gut.

C) It has lots of mobility and moves around in the peritoneal cavity with peristaltic movements. The greater omentum has considerable mobility and moves around in the peritoneal cavity with the peristaltic movements of the intestines. It also cushions the abdominal organs against injury and provides insulation against the loss of body heat. The mesentery holds the organs in place and attaches the jejunum and ileum to the abdominal wall. The mucosal layer acts as a barrier to prevent the entry of noxious substances and pathogenic organisms.

A speech therapist is performing a swallowing assessment on a 72-year-old man who has suffered a stroke 3 weeks ago. The man has been NPO (nothing by mouth) since his stroke, and the health care team is considering the introduction of oral food. The speech therapist is cueing the client to swallow to preclude either aspiration of food or pocketing of food in the sides of his mouth. The client most likely to have conscious control over which of the following processes listed below involved in swallowing? A) Initiation of primary peristalsis B) Moving the epiglottis back to cover the larynx C) Moving a bolus to the posterior wall of the pharynx D) Moving the bolus backward in the esophagus

C) Moving a bolus to the posterior wall of the pharynx Moving a bolus to the posterior wall of the pharynx is a component of the oral, or voluntary, phase of swallowing. Initiation of primary peristalsis, moving the epiglottis back to cover the larynx, and moving the bolus backward in the esophagus are all involuntary components of the pharyngeal and esophageal phases of swallowing.

A 51-year-old woman has been diagnosed with Cushing syndrome after a diagnostic workup that reveals cortisol hypersecretion. The nurse knows which of the following assessment findings would be inconsistent with her diagnosis? A) Increased blood pressure and decreased potassium levels B) A protruding abdomen and a "buffalo hump" on the back C) Poor stress management and hyperpigmentation D) A "moon face" and muscle weakness

C) Poor stress management and hyperpigmentation A low tolerance for stress and hyperpigmentation is associated with Addison disease and its consequent elevated levels of ACTH. High blood pressure, hypokalemia, buffalo hump, and moon face are all characteristics of the elevated steroid levels that denote Cushing syndrome.

A nurse who works on an oncology ward is providing care for a 68-year-old female patient with a diagnosis of lung cancer with bone metastases. The client is experiencing rapid weight loss and is exhibiting the signs and symptoms of malnutrition. The nurse would recognize that which of the following factors is most likely contributing to the client's malnutrition? A) Autoimmune responses associated with acute illness are inhibiting anabolism. B) Chronic hypoxia is precluding many of the aerobic processes required in body maintenance and repair. C) Protein mass is being lost from the liver and other organs, and the liver is synthesizing fewer serum proteins. D) Intestinal malabsorption is occurring as a result of tumor metastases.

C) Protein mass is being lost from the liver and other organs, and the liver is synthesizing fewer serum proteins. Ill individuals are prone to disruption in protein balance, in which protein breakdown exceeds protein rebuilding. Protein mass is lost from the liver, gastrointestinal tract, kidneys, and heart. As protein is lost from the liver, hepatic synthesis of serum proteins decreases and decreased levels of serum proteins are observed. Autoimmune processes, hypoxia, and malabsorption as a result of metastases are unlikely factors.

A 55-year-old man has been diagnosed with a gastroesophageal reflux disease (GERD), in which the function of his lower esophageal sphincter is compromised. Which of the following consequences of this condition is most likely to occur? A) Decreased absorption of ingested foods and fluids B) Impaired control of the gastric emptying rate C) Protrusion of the stomach or regurgitation of stomach contents into the esophagus D) Inappropriate release of gastric enzymes

C) Protrusion of the stomach or regurgitation of stomach contents into the esophagus Given that the role of the lower esophageal sphincter is to control the exchange of foods and fluids, a deficit is likely to allow the stomach contents, or the stomach itself, to protrude into the esophagus. Absorption is unlikely to be directly affected, and the sphincter is not responsible for controlling gastric emptying or enzyme secretion.

A male infant is brought into the clinic because of colic-like symptoms. The mother states he acts like something is hurting. After eating, he vomits most of the feeding and then assumes a fetal position. He is also not gaining weight. The nurse practitioner is thinking that he is displaying clinical manifestations of obstruction and may have which of the following medical diagnoses? A) Duodenal ulceration B) Constipation C) Pyloric stenosis D) Erosive esophagitis

C) Pyloric stenosis An example of obstruction is hypertrophic pyloric stenosis, which can occur in infants with an abnormally thick muscularis layer in the terminal pylorus. A defect in the lining of the first part of the small intestine (duodenal ulcer) is usually caused by an infection with a bacterium (germ) called H. pylori. When food is ingested and digested but not excreted, it forms a blockage in the colon. Regular bowel movements are needed in order for this not to occur. When bowel movements are irregular, constipation may result. This infant appears to be vomiting his stomach content; therefore, no BM is occurring since no food is being digested in the small intestine. Gastrin provides the major stimulus for gastric acid production. Its action on the lower esophageal sphincter protects the esophageal mucosa when gastric acid levels are elevated. If stomach acids reflux into the esophagus, acid irritation and inflammation cause extensive injuries to the esophagus.

A male client complaining of chronic cramping, bloating, and diarrhea has been determined to have a deficiency in brush border enzymes within his small intestine. Which of the following meals or snacks high in carbohydrates and protein will likely exacerbate the client's signs and symptoms? A) Grapefruit and prunes B) Tossed salad with an oil and vinegar dressing C) Roast beef and a baked potato D) Tortilla chips and guacamole

C) Roast beef and a baked potato Brush border enzymes are primarily responsible for the metabolism of carbohydrates and proteins, substances best exemplified by a baked potato and roast beef, respectively. Of the distracters, choice C has both high protein and high carbohydrate content. Distracters A, B, and D are not high in both carbohydrates and proteins.

A 21-year-old female is suspected of having inadequate function of her hypothalamic-pituitary-thyroid system. Her care provider is planning to inject thyrotropin-releasing hormone (TRH) and then measure her levels of TSH. Which of the following diagnostic tests is being performed? A) Suppression test B) Radioimmunoassay (RIA) test C) Stimulation test D) Metabolite excretion test

C) Stimulation test A stimulation test involves the introduction of an element that stimulates the production of another factor or hormone followed by measurement of that hormone. This is not the case in a suppression test, RIA test, or metabolite excretion test.

When explaining the role of liver Kupffer cells to a group of nursing students, which of the following statements about the function of these cells is most accurate? A) The primary function of Kupffer cells is to secrete bile. B) These cells are the functional unit of the liver and are responsible for all liver secretions. C) The cells are capable of removing and phagocytizing old and defective blood cells. D) The role of the Kupffer cells is to provide at least 50% of cardiac output each minute to each lobular of the liver.

C) The cells are capable of removing and phagocytizing old and defective blood cells. Kupffer cells are reticuloendothelial cells that are capable of removing and phagocytizing old and defective blood cells, bacteria, and other foreign material from the portal blood as it flows through the sinusoid. This phagocytic action removes enteric bacilli and other harmful substances that filter into the blood from the intestine. Small tubular channels in the liver secrete bile. The functional unit of the liver is lobule. Approximately 25% of blood per minute enters the liver through the hepatic artery.

A 51-year-old male has been diagnosed with alcohol-induced liver disease. He admits to the nurse providing his care that, "I know what the lungs do, and I know what the heart does, but honestly I have no idea what the liver does in the body." Which of the following statements would best underlie the explanation that the nurse provides? A) The liver is responsible for the absorption of most dietary nutrients as well as the production of growth hormones. B) The liver contributes to the metabolism of ingested food and provides the fluids that the GI tract requires. C) The liver metabolizes most components of food and also cleans the blood of bacteria and drugs. D) The liver maintains a balanced level of electrolytes and pH in the body and stores glucose, minerals, and vitamins.

C) The liver metabolizes most components of food and also cleans the blood of bacteria and drugs. The liver metabolizes protein, carbohydrate, and fat. As well, it metabolizes drugs and removes bacteria by Kupffer cells. Absorption of nutrients takes place in the intestines, and the liver does not produce the bulk of fluids secreted in the GI tract. The liver does not have a primary role in the maintenance of acid-base or electrolyte balance.

The nurse walks into a room and finds the patient forcefully expelling stomach contents into a wash basin. When documenting this occurrence, the nurse will use the term A) nauseous. B) retching. C) vomiting. D) expatriate.

C) vomiting. Vomiting or emesis is the sudden and forceful oral expulsion of the contents of the stomach. It is usually preceded by nausea. Nausea is a subjective and unpleasant sensation. Retching consists of rhythmic spasmodic movements of the diaphragm, chest wall, and abdominal muscles. It usually precedes or alternates with periods of vomiting. Expatriate means to banish or withdraw.

A male infant who is 48 hours postpartum is undergoing phototherapy for the treatment of jaundice and accompanying high levels of bilirubin. Place the following components of the production of bilirubin in the correct chronological order. Use all the options. A) Conjugated bilirubin B) Urobilinogen C) Red blood cells D) Biliverdin E) Free bilirubin

C, D, E, A, B Hemoglobin from the red blood cell is broken down to form biliverdin, which is rapidly converted to free bilirubin. Inside the hepatocytes, free bilirubin is converted to conjugated bilirubin. Conjugated bilirubin is secreted as a constituent of bile, and in this form, it passes through the bile ducts into the small intestine. In the intestine, approximately one half of the bilirubin is converted into a highly soluble substance called urobilinogen by the intestinal flora.

An endocrinologist is providing care for a 30-year-old male who has lived with the effects of increased levels of growth hormone (GH). Which of the following teaching points about the client's future health risks is most accurate? A) "It's not unusual for high GH levels to cause damage to your hypothalamus." B) "GH excess inhibits your pancreas from producing enough insulin." C) "The high levels of GH that circulate in your body can result in damage to your liver." D) "When your pituitary gland is enlarged, there's a real risk that you'll develop some sight deficiencies."

D) "When your pituitary gland is enlarged, there's a real risk that you'll develop some sight deficiencies." GH excess is associated with tumor formation and consequent compression of cranial nerves responsible for vision. Damage to the hypothalamus and liver is not common sequelae. While the beta cells of the pancreas can "burn out," the primary effect of excess GH is to increase insulin secretion.

Which of the following clients would be most reasonably expected to have the highest basal metabolic rate (BMR), assuming none is obese or malnourished? A) A 22-year-old man, 69 inches tall, who has a sedentary lifestyle B) A 47-year-old woman, 65 inches tall, who swims 1 mile four times a week C) A 29-year-old woman, 61 inches tall, who is pregnant and on bed rest D) A 60-year-old man, 72 inches tall, who is recovering from heart surgery

D) A 60-year-old man, 72 inches tall, who is recovering from heart surgery Although age, sex, physical state, and pregnancy are contributing factors, variations in BMR among individuals are chiefly related to skeletal muscle mass and body size. Therefore, regardless of health and fitness status, the tallest and heaviest person in any sample will probably have the highest BMR.

After receiving change-of-shift report about the following four patients, which patient should the nurse assess first? A) A 22-year-old admitted with SIADH who has a serum sodium level of 130 mEq/L B) A 31-year-old who has iatrogenic Cushing syndrome with a capillary blood glucose level of 204 mg/dL C) A 53-year-old who has Addison disease and is due for a scheduled dose of hydrocortisone (Solu-Cortef) D) A 70-year-old returning from PACU following partial thyroidectomy who is extremely agitated, has an irregular pulse rate of 134, and has an elevated temperature of 103.2°F

D) A 70-year-old returning from PACU following partial thyroidectomy who is extremely agitated, has an irregular pulse rate of 134, and has an elevated temperature of 103.2°F Manipulation of a hyperactive thyroid gland during thyroidectomy can cause thyroid storm. It is manifested by very high fever, extreme cardiovascular effects (tachycardia, HF, angina), and severe CNS effects (agitation, restlessness, and delirium). Answer choice A refers to normal sodium levels. Answer choice B refers to high blood glucose level but not critical level. Answer choice C refers to lower priority. It is always preferred to give medications in a timely manner; however, thyroid storm signs and symptoms are the priority for this group of patients.

During a humanitarian trip to an underdeveloped country, a medical student is assessing a 6-year-old male who has a protuberant abdomen, dry hair, and wrinkled skin. The child's heart rate is 59 beats/minute, blood pressure 89/50, and temperature 95.2°F (35.1°C). What is the most likely etiology of the child's health problems? A) A diet lacking in fat-soluble vitamins B) Fluid and electrolyte imbalances secondary to low -carbohydrate intake C) A diet that is low or high in carbohydrates but low in fat D) A diet deficient in both protein and calories

D) A diet deficient in both protein and calories The child's presentation is typical of marasmus, a diagnosis caused by deficiencies in protein and calorie intake.

While on tour, a 32-year-old male musician has presented to the emergency department of a hospital after a concert complaining of severe and sudden abdominal pain. He admits to a history of copious alcohol use in recent years, and his vital signs include temperature 46.8°C (101.8°F), blood pressure 89/48 mm Hg, and heart rate 116 beats/minute. Blood work indicates that his serum levels of C-reactive protein, amylase, and lipase are all elevated. Which of the following diagnoses would the care team suspect first? A) Hepatitis C B) Cholecystitis C) Liver cirrhosis D) Acute pancreatitis

D) Acute pancreatitis Alcohol use, fever, hypotension, and tachycardia are often associated with pancreatitis, as are elevated serum amylase and lipase levels. These enzymes would unlikely rise in cases of hepatitis, cholecystitis, or cirrhosis. The precise mechanisms whereby alcohol exerts its action are largely unknown. The capacity for oxidative and nonoxidative metabolism of ethanol by the pancreas and the harmful by-products that result have been related to the disease process. Hepatitis C has an incubation period. Most adults who acquire the infection usually are asymptomatic. Jaundice is uncommon. Direct measurement of HCV in the serum remains the most accurate test for infection. Cirrhosis represents the end stage of chronic liver disease. The end result is liver failure that affects many organs. The patients usually have anemia, thrombocytopenia, endocrine disorders, skin lesions, azotemia and renal failure, and hepatic encephalopathy.

A patient is admitted to the hospital in adrenal crisis 1 month after a diagnosis of Addison disease. The nurse knows which of the following clinical manifestations would support this diagnosis? A) Hyperactive deep tendon reflexes and slow, shallow breathing B) Cerebral spinal fluid leakage and impaired swallowing C) Irregular heart rate and decreased temperature D) Change in the level of consciousness and profound hypotension

D) Change in the level of consciousness and profound hypotension Acute adrenal crisis is a life-threatening situation. Exposure to even a minor illness or stress can cause a client with Addison disease to develop nausea, vomiting, muscular weakness, hypotension, dehydration, and vascular collapse (which causes a change in LOC). Hemorrhage (low BP) can be caused by septicemia, adrenal trauma, anticoagulant therapy, adrenal vein thrombosis, or adrenal metastases. A hyperactive reflex may indicate disease of the pyramidal tract above the level of the reflex arc being tested. Generalized hyperactivity of DTRs may be caused by hyperthyroidism. Any tear or hole in the membrane that surrounds the brain and spinal cord (dura) can allow the fluid that surrounds those organs to leak. This fluid is called the cerebrospinal fluid (CSF). When it leaks out, the pressure around the brain and spinal cord drops. Causes of leakage through the dura include certain head, brain, or spinal surgeries; head injury; placement of tubes for epidural anesthesia or pain medications; or lumbar puncture. Irregular heart rates (arrhythmias) may be caused by many different factors, including coronary artery disease; electrolyte imbalances in your blood (such as sodium or potassium); changes in your heart muscle; or injury from a heart attack.

A 20-year-old male college student has recently finished a Thanksgiving dinner of heroic proportions while home for the holiday weekend. Which of the following phenomena would most likely have produced his sensation of satiety? A) Stretch receptors in the stomach and small intestine signal the feeding center in the medulla. B) Increased levels of leptin stimulate a decrease in appetite by way of the vagus nerve. C) The breakdown of products of lipids such as ketoacids produces a decrease in appetite. D) Cholecystokinin and glucagon-like peptide-1 suppress the hunger impulse.

D) Cholecystokinin and glucagon-like peptide-1 suppress the hunger impulse. The presence of fat in the duodenum and nutrients in the small bowel results in the release of cholecystokinin and glucagon-like peptide-1, respectively, which suppress the feeding center located in the hypothalamus. Leptin and ketoacids are associated with the intermediate and long-term regulation of food intake rather than the short-term control that would signal the end of a meal or snack.

A 24-year-old woman undergoing a premarital screening test is found to have elevated levels of AST, ALT, and IgG, but no antibody-specific markers for viral hepatitis. A liver biopsy reveals inflammation and cellular damage. Which of the following treatments is most likely to be effective for her? A) Lamivudine B) Peginterferon and ribavirin C) Interferon alfa-2b D) Corticosteroids and immunosuppressant drugs

D) Corticosteroids and immunosuppressant drugs This woman's hepatitis is probably caused by an autoimmune disorder rather than a virus. Lamivudine, peginterferon/ribavirin, and interferon alfa-2b are all antiviral agents. Autoimmune hepatitis is a chronic, usually progressive, inflammatory disease of the liver. Corticosteroids are the mainstay of treatment and have been shown to produce remission. Azathioprine, an immunosuppressant medication, is sometimes used along with prednisone. Using both medications may allow you to take a smaller dose of prednisone, reducing its side effects. Most people with this disease require long-term maintenance treatment.

Following the identification of low levels of T3 and T4 coupled with the presence of a goiter, a 28-year-old female has been diagnosed with Hashimoto thyroiditis. In light of this diagnosis, which of the following assessment results would constitute an unexpected finding? A) The presence of myxedema in the woman's face and extremities B) Recent weight gain despite a loss of appetite and chronic fatigue C) Coarse, dry skin and hair with decreased sweat production D) Increased white cell count and audible crackles on chest auscultation

D) Increased white cell count and audible crackles on chest auscultation An increased white cell count and the presence of adventitious fluid in the lungs are not classic findings associated with hypothyroidism. Myxedema, weight gain, lethargy, and dry skin and nails are commonly associated with low levels of thyroid hormones.

A 79-year-old male resident of a long-term care facility has contracted Clostridium difficile and is experiencing consequent diarrhea. Auscultation of the man's abdomen indicates hyperactive bowel sounds. What process in the man's small intestine is most likely accompanying his current status? A) Pathogenic microorganisms are causing dilation of his small intestine, increasing motility. B) Segmentation waves have become more frequent as a result of his infection. C) Intestinal stasis brought on by infection is preventing his small intestine from sufficiently slowing the rate of motility. D) Inflammation is accompanied by an increase in peristaltic movements of his small intestine.

D) Inflammation is accompanied by an increase in peristaltic movements of his small intestine. Inflammation of the small intestine is accompanied by an increase in motility, an effect that is the result of increased peristaltic waves. Segmentation waves are responsible primarily for mixing rather than moving food. Neither dilation nor inadequate slowing of passage contents is responsible for the increase in motility.

Which of the following statements best communicates the nature of movements in the colon? A) The internal and external anal sphincters control the movement of feces into and through the colon. B) Haustrations move the colon contents along with 2- to 3-minute rest periods between movements. C) Segmentation waves ensure that all surfaces of the feces are exposed to the colon surface. D) Large segments of the colon contract as a unit for around 30 seconds.

D) Large segments of the colon contract as a unit for around 30 seconds. The colon, unlike the small intestine, moves contents by the simultaneous contraction of large segments. The anal sphincters control defecation rather than movements within the colon, and haustrations perform mixing, not motility. Segmentation waves are present in the small intestine.

A 20-year-old woman has visited her family physician due to occasional bouts of bloody diarrhea over the past several weeks, a phenomenon that she experienced 2 years prior as well. Her physician has diagnosed her with ulcerative colitis based on her history and visualization of the affected region by colonoscopy and sigmoidoscopy. Which of the following pathophysiological phenomena is most likely to underlie the client's health problem? A) Fissures and crevices developing in the mucosa that are seen as a characteristic "cobblestone" appearance B) Erosion of the endothelial lining of the distal small intestine by a combination of genetic, autoimmune, and environmental factors C) Compromise of the mucosal layer of the large intestinal surface by the effects of H. pylori D) Mucosal hemorrhages that have developed into crypt abscesses, which have in turn necrotized and ulcerated

D) Mucosal hemorrhages that have developed into crypt abscesses, which have in turn necrotized and ulcerated The etiology and course of ulcerative colitis involves mucosal hemorrhages developing into crypt abscesses, with consequent necrosis and ulceration. "Cobblestone" appearance of intestinal mucosa is associated with Crohn disease. Ulcerative colitis is confined to the colon and rectum, and H. pylori is not commonly implicated in the etiology.

A 77-year-old woman has been admitted to hospital following several weeks of increasing fatigue. On observation, she is pale, and blood work indicates she has low hemoglobin and red cell counts. Stool tests for occult blood are positive, and following endoscopy, she has been diagnosed with an upper GI bleed that has been shown to originate in her stomach. She admits to regularly exceeding the recommended doses of nonsteroidal anti-inflammatory drugs (NSAIDs) in an effort to control her rheumatoid arthritis. Which of the following phenomena is most likely responsible for her present health problems? A) NSAIDs increase the gastric production of gastrin, increasing gastric secretions and lowering stomach pH. B) Drugs such as NSAIDs increase the H+ levels and thus decrease gastric pH, resulting in insult to the stomach lining. C) NSAIDs, aspirin, and other drugs increase prostaglandin synthesis, resulting in disruption of cellular structures lining the stomach. D) NSAIDs can disrupt the permeability of the gastric mucosa, causing hydrogen ions to accumulate in the mucosal cells of the lining

D) NSAIDs can disrupt the permeability of the gastric mucosa, causing hydrogen ions to accumulate in the mucosal cells of the lining NSAIDs damage the mucosal barrier, allowing hydrogen ions to damage cells of the stomach lining. They do not directly increase gastrin production or H+ levels, and decreased, not increased, prostaglandin synthesis would potentially compromise the gastric surfaces.

A 48-year-old woman has been diagnosed with extrahepatic cholestasis following a thorough history, ultrasound, and blood work. Which of the following symptoms most likely caused her to seek medical treatment, and what consequence to her health problem would the medical team anticipate? A) Complaints of lower flank pain with consequences of impaired fat metabolism B) Anorexia with consequences of impaired drug metabolism and blood filtration C) Skin xanthomas (focal accumulations of cholesterol) with consequences of increased risk of bleeding due to deficient clotting factors D) Pruritus with consequences of deficient levels of fat-soluble vitamins

D) Pruritus with consequences of deficient levels of fat-soluble vitamins Pruritus is the most common symptom of cholestasis, and deficiencies in fat-soluble vitamins such as A, D, and K are frequent. Flank pain is not a noted complaint, and bile does not contribute to drug metabolism, blood filtration, or clotting factor production.

A 43-year-old male who is 5′10″ tall and weighs 216 lb has been informed by his nurse practitioner that his body mass index (BMI) is 31. Which of the following clinical conclusions based on these data would his nurse be most justified in rejecting? A) Further investigation of his nutritional status is needed to supplement the BMI value. B) The client faces an increased risk of type 2 diabetes and hyperlipidemia. C) He is classified as being obese, likely as the result of the interplay of genetic and lifestyle factors. D) The client is borderline obese but is not yet at the point of significantly increased risks to health.

D) The client is borderline obese but is not yet at the point of significantly increased risks to health. A BMI of 31 is classified as obese, and the client faces a risk of hypertension, hyperlipidemia, type 2 diabetes, coronary heart disease, and other health problems. While BMI is a valid instrument, other data sources are needed to supplement this value clinically. Obesity is considered to be an outcome of a variety of factors including heredity and lifestyle.

Which of the following statements best captures the relationship between the hypothalamus and the pituitary gland as it relates to endocrine function? A) The hypothalamus directly measures the levels of most hormones throughout the body and inhibits or stimulates the pituitary accordingly. B) The pituitary gland coordinates and dictates the release of hormones from the hypothalamus that act on their intended target cells. C) The pituitary gland and hypothalamus have two-way communication that mediates the signals from neuronal inputs. D) The hypothalamus receives input from numerous sources throughout the body and directs the pituitary to then control many target glands and cells.

D) The hypothalamus receives input from numerous sources throughout the body and directs the pituitary to then control many target glands and cells. The hypothalamus can be viewed as a bridge by which signals from multiple systems are relayed to the pituitary gland. The hypothalamus collects data from sources throughout the body rather than directly measuring levels, and communication normally flows from the hypothalamus to the pituitary.

A client with a history of an endocrine disorder exhibits signs and symptoms of hormone deficiency. Which of the following processes would the client's care team most likely rule out first as a contributing factor? A) The client's target cells lack sufficient receptors for the hormone in question. B) Hormone production is sufficient, but affinity on the part of the target cells is lacking. C) The process of down-regulation has resulted in decreased hormone sensitivity. D) Up-regulation has increased the sensitivity of the body to particular hormone levels.

D) Up-regulation has increased the sensitivity of the body to particular hormone levels. Up-regulation is a response to low hormone levels in which the number of receptors increases. As such, it would not likely result in signs and symptoms of deficiency but is rather a compensatory mechanism that counters a deficiency. Insufficient numbers of receptors, low affinity, and down-regulation could all contribute to signs and symptoms of a hormone deficiency.

Growth hormone (GH) secretion is inhibited by A) hypoglycemia. B) starvation. C) heavy exercise. D) obesity.

D) obesity. GH is inhibited by increased glucose levels, free fatty acid release, cortisol, and obesity. It is stimulated by hypoglycemia, fasting starvation, increased blood levels of amino acids, and stress conditions such as trauma, excitement, emotional stress, and heavy exercise.

Place the following components of the gastrointestinal tract in the chronological order that a bolus of food would pass through them. Use all the options. A) Ileum B) Pylorus C) Jejunum D) Hiatus E) Cecum F) Duodenum

D, B, F, C, A, E Ingested food and fluids enter the stomach through the hiatus, exit through the pylorus, and pass through the three subdivisions of the small intestine: the duodenum, jejunum, and ileum. The cecum is a component of the large bowel.


Related study sets

NRS 3025: Exam #1 Practice Questions

View Set

Personal Finance Question Ch 1-7

View Set

Business Management & Admin MASTER KEY

View Set

Chapter 38: Agents to Control Blood Glucose Levels

View Set

Managerial Accounting Test 1 Concepts

View Set